Đến nội dung

xuanhoan23112002 nội dung

Có 95 mục bởi xuanhoan23112002 (Tìm giới hạn từ 29-03-2020)



Sắp theo                Sắp xếp  

#642083 $\boxed{\text{Chuyên Đề}}$ Phương trình vô tỉ - Hệ phương...

Đã gửi bởi xuanhoan23112002 on 25-06-2016 - 09:09 trong Phương trình, hệ phương trình và bất phương trình

giúp mình bài này với: $x^4+1=5x(x^2-2)$




#662621 $\boxed{\text{Chuyên Đề}}$ Phương trình vô tỉ - Hệ phương...

Đã gửi bởi xuanhoan23112002 on 21-11-2016 - 17:24 trong Phương trình, hệ phương trình và bất phương trình

Các bạn suy nghĩ bài này xem:

giải hệ phương trình:

     5x2+ 2y2+ z=2

     xy+ yz+ xz =1




#662624 $\boxed{\text{Chuyên Đề}}$ Phương trình vô tỉ - Hệ phương...

Đã gửi bởi xuanhoan23112002 on 21-11-2016 - 17:36 trong Phương trình, hệ phương trình và bất phương trình

Giải phương trình  x2 - 5x + 36 = 8 $\sqrt{3x+4}$

DKXD:x>=-4/3

Cậu bình phương cả 2 vế lên:

(x2-5x+36)2=64(3x+4)

x4-10x3+ 97x- 552x+1040=0

(x-4)2(x- 2x + 65)=0

x=4 (thỏa man DKXD)




#662648 $\boxed{\text{Chuyên Đề}}$ Phương trình vô tỉ - Hệ phương...

Đã gửi bởi xuanhoan23112002 on 21-11-2016 - 20:49 trong Phương trình, hệ phương trình và bất phương trình

Các bài làm hầu như không ghi ĐKXĐ vào, các bạn chú ý nhé. (Nhớ trích dẫn đề nữa)

 

Giải pt:

19) $\sqrt{2x^2+8x+6}+\sqrt{x^2-1}=2x+2$

 

20) $\sqrt[3]{24+x}+\sqrt{12-x}=6$

 

21) $(x+1)(x+4)=5\sqrt{x^2+5x+28}$

 

22) $4\sqrt{(4-x)(2+x)}=x^2-2x-12$

 

23) $\sqrt{3x-2}+\sqrt{x-1}=4x-9+2\sqrt{3x^2-5x+2}$

 

24) $5\sqrt{x}+\frac{5}{2\sqrt{x}}=2x+\frac{1}{2x}+4$

 

25) $x^2-2x=2\sqrt{2x-1}$

 

26) $x^3+1=2\sqrt[3]{2x-1}$

câu 21:DKXD: với mọi x

sau đó đặt cận vệ phải là a(a>=0) tôi biểu diễn về trại theo a và phân tích thành tích là xong




#662739 Chuyên đề : Làm mạnh BĐT CôSy

Đã gửi bởi xuanhoan23112002 on 22-11-2016 - 20:51 trong Bất đẳng thức và cực trị

Tìm giá trị nhỏ nhất của 

P=$\fn_jvn \frac{a}{bc}$+$\fn_jvn \frac{2b}{ca}$+$\fn_jvn \frac{5c}{ab}$

trong đó a2+b2+c2=6




#668372 $\boxed{\text{Chuyên Đề}}$ Phương trình vô tỉ - Hệ phương...

Đã gửi bởi xuanhoan23112002 on 15-01-2017 - 10:34 trong Phương trình, hệ phương trình và bất phương trình

Giải phương trình này: $\sqrt{1-\frac{1}{x}}+\sqrt{x^2-x}=2$




#668399 MỘT SỐ PHƯƠNG PHÁP GIẢI TOÁN TỔ HỢP THCS

Đã gửi bởi xuanhoan23112002 on 15-01-2017 - 12:32 trong Toán rời rạc

ko vi no ch

 

Bài 15 : Một bạn cờ quốc tế $8\times 8$  . Hỏi rằng quân mã có thể đi nước đầu tiên từ ô dưới cùng bên trái và kết thúc ở ô trên cùng bên phải không ? Với điều kiện nó phải đi qua tất cả các ô trên bàn cờ và mỗi ô chỉ đi qua đúng một lần. 

ko vi quan ma ko the di het ban co




#669654 MỘT SỐ PHƯƠNG PHÁP GIẢI TOÁN TỔ HỢP THCS

Đã gửi bởi xuanhoan23112002 on 24-01-2017 - 06:11 trong Toán rời rạc

Trên mặt phẳng cho 5 điểm có tọa độ nguyên trong đó không có 3 điểm nào thẳng hàng .Chứng minh rằng trong số các tam giác tạo thành từ 5 điểm đã cho có ít     nhất 3 tam giác có diện tích nguyên




#669686 Trong 100.000.000 số hạng đầu tiên của dãy Fibonacci, có tồn tại hay...

Đã gửi bởi xuanhoan23112002 on 24-01-2017 - 13:57 trong Tổ hợp và rời rạc

Ký hiệu $F_{n}$ là số hạng thứ $n$ của dãy Fibonacci

Chia mỗi số $F_{n}$ cho $10^4$ đc các số dư là $r_{n}$ (mod $10^4$) với $r_{i}=\overline{0,9999}$

Xét tất cả các cặp có thể có của các số nguyên từ $0$ đến $9999$ có $10^8$ cặp $(r_{0};r_{0});(r_{0};r_{1});(r_{1};r_{2});...$

Giả sử ko có cặp nào là $(0;0)$ thì các cặp trên phải có 2 cặp trùng nhau theo nguyên lý Dirichlet, gsử:

  $r_{n}=r_{p}$ và $r_{n+1}=r_{p+1}$ ($n>p$)

$\Rightarrow r_{n-1}=r_{p-1} \Rightarrow r_{n-2}=r_{p-2} \Rightarrow ... \Rightarrow r_{1}=r_{n-p+2}=r_{n-p+1} \Rightarrow r_{n-p+1}=r_{n-p+2} \Rightarrow r_{n-p}=0$

Vậy tồn tại $1$ số $F_{n}$ chia hết cho $10^4$ hay có tận cùng là $4$ chữ số $0$

Mình không hiểu bài này lắm, gửi lại cho mình đi.




#683636 Đề thi toán vòng 2 thpt chuyên Lê Hồng Phong Nam Định 2017-2018

Đã gửi bởi xuanhoan23112002 on 08-06-2017 - 09:52 trong Tài liệu - Đề thi

moi nguoi hom nay co ket qua LHP day hoi hop qua hi vong do co vu cho minh nhe




#683639 Đề thi toán vòng 2 thpt chuyên Lê Hồng Phong Nam Định 2017-2018

Đã gửi bởi xuanhoan23112002 on 08-06-2017 - 09:54 trong Tài liệu - Đề thi

minh nghi bai to hop cac ban can chung minh co 1 tam giac co 3 dinh la 3 trong cac diem da cho va canh cua tam giac nho hon 1 thi dung cong thuc tinh dien tich bang sin60 se ra ngay




#683641 Đề thi toán vòng 2 thpt chuyên Lê Hồng Phong Nam Định 2017-2018

Đã gửi bởi xuanhoan23112002 on 08-06-2017 - 09:57 trong Tài liệu - Đề thi

Mình thấy cách giải của bạn ddang00 không hợp lí lắm. Như cách giải thích của anh IHateMath thì có vẻ cách của bạn chưa đúng hơn nữa nếu làm như vậy con số $\frac{3\sqrt{3}}{4}cm^{2}$ không có ý nghĩa cho lắm.

 

Cách giải của mình như sau:

 

attachicon.gifScreenHunter_35 May. 30 14.25.jpg

Ta chia tứ giác $ABCD$ thành $16$ tứ giác nội tiếp trong đường tròn bán kính $1$ như hình trên bằng cách lấy các trung điểm của cạnh tứ giác $ABCD$ và làm thế 1 lần nữa với $4$ tứ giác vừa được chia ra.

Theo nguyên lí $Dirichlet$ thì tồn tại $3$ điểm trong $33$ đã cho cùng thuộc $1$ tứ giác trong $16$ tứ giác vừa được chia ra

$3$ điểm này thuộc hình tròn bán kính bằng $1$. Ta sẽ chứng minh $3$ điểm này là $3$ điểm cần tìm.

attachicon.gifScreenHunter_36 May. 30 14.38.jpg

Gọi $3$ điểm này là $E,F,G$

Xảy ra $3$ trường hợp:

TH1 3 điểm này không nằm trên đường tròn. Vẽ $EF$ cắt $(I)$ tại $M$. Đường thẳng $EG$ cắt $(I)$ tại $N,K$.

Dễ thấy $S_{EFG}< S_{MNK}$. Mà ta lại có diện tích của 1 tam giác bất kì nội tiếp đường tròn không quá diện tích tam giác đều nội tiếp đường tròn đó. Dễ tính được diện tích tam giác đều nội tiếp đường tròn bán kính $1$ là $\frac{3\sqrt{3}}{4}cm^{2}$

Suy ra $S_{EFG}< S_{MNK}\leq \frac{3\sqrt{3}}{4}cm^{2} $

TH2 Tồn tại ít nhất $1$ điểm trong $3$ điểm nằm trên đường tròn.

Vẽ như TH1 và giải như TH1

TH3 3 điểm này nằm trên đường tròn. Giải như TH1 thì $S_{EFG}\leq  \frac{3\sqrt{3}}{4}cm^{2}$

 

Như vậy ta có điều phải chứng minh

sao ban biet 16 tu giac deu noi tiep




#685108 Đề thi toán chuyên - chuyên KHTN ĐHQG HÀ Nội vòng 2 2017

Đã gửi bởi xuanhoan23112002 on 20-06-2017 - 09:12 trong Tài liệu - Đề thi

Câu II 2.

Từ giả thiết ta có được $\frac{1}{(a+1)(b+1)}+\frac{1}{(c+1)(b+1)}+\frac{1}{(a+1)(c+1)}=1$

 Đặt $a+1=\frac{\sqrt{3}}{x}, b+1=\frac{\sqrt{3}}{y},c+1=\frac{\sqrt{3}}{z}$

Giả thiết trở thành $xy+yz+zx=3$ và

$P= \sqrt{3} ( \frac{1}{\frac{3}{x}+x} +\frac{1}{\frac{3}{y}+y} +\frac{1}{\frac{3}{z}+z})$

   $= \sqrt{3} (\frac{x}{x^{2}+3}+\frac{y}{y^{2}+3}+\frac{z}{z^{2}+3})$

Sử dụng giả thiết ta có

  $P=\sqrt{3}( \frac{x}{(x+y)(x+z)}+ \frac{y}{(x+y)(y+z)}+ \frac{z}{(z+y)(x+z)})$

    $=\sqrt{3}( \frac{2(xy+yz+zx)}{(x+y)(y+z)(z+x)})$

Mặt khác $(x+y)(y+z)(z+x) \geq \frac{8}{9}(x+y+z)(xy+yz+zx) \geq \frac{8}{3}(xy+yz+zx)$

Suy ra $P \leq \sqrt{3}\frac{3}{4}= \frac{3\sqrt{3}}{4}$

Đẳng thức xảy ra khi $a=b=c = \sqrt{3}-1$                                                      

mình có cách khác nhanh hơn đặt a+1=x;b+1=y;c+1=z thì bài toán sẽ tự nhiên hơn và ứng dụng được cả giả thiết




#688311 58th IMO 2017

Đã gửi bởi xuanhoan23112002 on 22-07-2017 - 08:38 trong Thi HSG Quốc gia và Quốc tế

moi nguoi nghi ra huong giai quyet bai 3 chua




#691945 $P=\frac{(x+1)^{2}(y+1)^{2}}{(x-...

Đã gửi bởi xuanhoan23112002 on 31-08-2017 - 13:28 trong Bất đẳng thức và cực trị

Bài này khá dễ giống đề thi lớp 10 ở VĨnh Phúc,. Bạn có thể tham khảo, hãy like cho mình nhé




#703960 2(ab+bc+ca)+$\frac{1}{ab}+\frac{1...

Đã gửi bởi xuanhoan23112002 on 19-03-2018 - 23:21 trong Bất đẳng thức và cực trị

Su dung pp pqr
Dat p=a+b+c=3

q=ab+bc+ca

r=abc,r<=1

BDT tuong duong 2q+3/r>=9

Hay 2qr+3>=9r

Ma q>=3*can(r)( do q^2>=3pr)

Dua ve bpt an r giai voi chu y r<=1




#704021 2(ab+bc+ca)+$\frac{1}{ab}+\frac{1...

Đã gửi bởi xuanhoan23112002 on 20-03-2018 - 22:08 trong Bất đẳng thức và cực trị

bạn bị ngược dấu hay sao ấy

nguoc dau cho nao




#704835 TOPIC thảo luận, trao đổi toán thi học sinh giỏi khối 10,11 .

Đã gửi bởi xuanhoan23112002 on 03-04-2018 - 22:54 trong Chuyên đề toán THPT

Đóng góp bài này cho topic

Cho a,b,c,d  thỏa mãn a+b+c+d=6 và a2+b2+c2+d2=12

cmr 48 >= 4(a3+b3+c3+d3) -(a4+b4+c4+d4) >= 36

Lời giải:
4(a3+b3+c3+d3) - (a4+b4+c4+d4) = -((a-1)4+(b-1)4+(c-1)4+(d-1)4)+52
Đặt x=a-1, y=b-1, z=c-1, t=d-1.
Bất đẳng thức cần chứng minh tương đương với:
 16>= x4+y4+z4+t4>=4
Ta cũng có: x2+y2+z2+t2=4
Từ đây áp dụng bất đẳng thức AM-GM ta có điều phải chứng minh(Q.E.D) 
Đẳng thức xảy ra tại (a,b,c,d) = (3,1,1,1) và (2,2,2,0) và các hoán vị của nó



#704920 Lập phương trình đt d qua M

Đã gửi bởi xuanhoan23112002 on 04-04-2018 - 22:14 trong Phương pháp tọa độ trong mặt phẳng

Cho điểm M(2;1). Lập d đi qua M cắt Ox, Oy tại A, B sao cho khoảng cách từ O đến d là max

d:ax+by-2a-b=0

(a,b khác 0)

Từ đây ta tìm được tọa độ giao điểm của (d) với Ox, Oy theo a, b

Áp dụng hệ thức lượng trong tam giác vuông tính được khoảng cách từ O đến d theo a,b

Tìm max của giá trị.




#704997 tìm giá trị của x để a nguyên

Đã gửi bởi xuanhoan23112002 on 05-04-2018 - 21:04 trong Đại số

PP làm: lấy tử chia mẫu được phần nguyên và phân số rồi sử dụng tính chia hết




#705136 Chứng minh rằng với mọi số thực dương $a,b>2$ thì $2^a-1...

Đã gửi bởi xuanhoan23112002 on 06-04-2018 - 22:59 trong Số học

Bài toán sai khi a chia hết cho b 

Nếu a không chia hết cho b. Đặt a=bq+r(0<r<b)

Sử dụng phản chứng để suy ra 2r - 1 chia hết cho 2b - 1 (điều này vô lí do 0<2r - 1<2b - 1)

Từ đó ta có điều phải chứng minh.




#705137 chứng minh

Đã gửi bởi xuanhoan23112002 on 06-04-2018 - 23:24 trong Số học

chứng minh rằng với mọi n$\epsilon$$\mathbb{Z}$ thì $n^{5}$ và n có chữ số tận cùng giống nhau 

n5 - n =n(n-1)(n+1)(n2+1)

Dễ dàng chứng minh n5-n chia hết cho 2

Với n chia hết cho 5, chia 5 dư 1 hoặc 4 thì n5-n chia hết cho 5

Với n chia 5 dư 2 hoặc 3 thì n2+1 chia hết cho 5

Như vậy ta có n5-n chia hết cho 5, n5-n chia hết cho 2 và gcd(5,2)=1

Nên n5-n chia hết cho 10

Hay n5 và n có chữ số tận cùng giống nhau




#705140 chứng minh

Đã gửi bởi xuanhoan23112002 on 06-04-2018 - 23:36 trong Số học

Lời giải nhanh nhất cho bài toán này là cách áp dụng định lý Fermat nhỏ:

x- x chia hết cho 5

Dễ dàng chứng minh x5 - x chia hết cho 2

Như vậy ta cũng có điều phải chứng minh




#705203 Trong một giải đấu bóng đá có 10 đội tham gia theo thể thức mỗi đội đều gặp đ...

Đã gửi bởi xuanhoan23112002 on 08-04-2018 - 08:27 trong Tổ hợp và rời rạc

Ta chứng minh bài toán bằng phản chứng( cả 1 và 2 đều không xảy ra)

Gọi 10 đội bóng là a(i là số tự nhiên và i chạy từ 1 đến 10)

Giả sử a10 là đội bóng có số trận thua nhiều nhất

Khi đó nếu tồn tại giá trị i từ 1 đến 9 mà a10 thang ai thì tất cả cả đội bóng mà a10 thua thì ai cũng thua (vô lí do a10 có số trận thua nhiều nhất)

Suy ra a10 thi đấu với các đội còn lại chỉ có thể hòa hoặc thua

Mà theo gia sư điều kiện 2 không xảy ra nên a10 thua ít nhất 7 đội là aj (j chạy từ 1 đến 7)

Lập luận tương tự như trên với a7 là đội có số trận thua nhiều nhất trong 7 đội trên thì a7 phải thừa ít nhất 4 đội giả sử là: a1, a2, a3, a4.

Lập luận tương tự như trên với a4 là đội có số trận thua nhiều nhất trong 4 đội trên thì a4 phải thừa ít nhất 1 đội giả sử là: a1

Như vậy ta tìm được 4 đội: a1, a4, a7, a10, lập thành 4 đội thỏa mãn điều kiện 1( mâu thuẫn với giả sử)

Do đó giả sử sai. Ta có điều phải chứng minh




#705205 Tìm a? đề hàm số có đạo hàm tại x=-1

Đã gửi bởi xuanhoan23112002 on 08-04-2018 - 08:32 trong Hàm số - Đạo hàm

Ta phải xét tính liên tục của hàm số và đặt điều kiện đạo hàm trái = đạo hàm phải bạn nhé